Exploring the Paradoxical Light Clock Problem in Special Relativity Analysis

In summary: No, angles are comparative, it just means that the angle between the emitted light and the observer is the same as the angle between the emitted light and the diagram.
  • #106
OK, I think that we are getting somewhere:
altergnostic said:
[little mutual misunderstanding ...]
Do it like the train and embankment problem and simply ditch the angle. You are given the primed distance between AB and the primed time for event B, you don't need anything else.
A bullet doesn't have the same speed in both frames; I'll stick to the topic instead, which allows much better clarity.
[..] The light between the train and embankment acts the same as light between beam and detector, and it is this light that we directly observe that we know moves at c. It is this light that brings us the coordinates for the beam.
I already illustrated that it is not necessarily to use that light for the time data. So, if you mean that it is light or radio waves or other means that brings us the clock synchronisation, then I completely agree.
[..] Actually this hasn't been answered at all, but I already concluded that light that is moving in any direction other than directly at us doesn't have to follow any constancy, since we can't be considered neither source nor observer in that case.
In post #8 I gave you the answer (in the link). However, you did not comment on it.
[EDIT: In fact, my answer took care of the spatial rotation misconception]
[..]You showed it can be done with a rigorous setup, but I still contend the numbers are different from current diagrams.
Of course, we narrowed down the problem as stemming from a calculation error - either made by "all" textbooks and students, or made by you. My purpose was (and still is) to get rid of the bug, after which you can be an asset to this forum - and you will feel better. :smile:
[..]I actually learned a lot from your scattering setup
Thanks - such feedback is helpful to remain motivated with this voluntary work! :tongue2:
so it is not like I changed the subject, I only went where the discussion led me. [..]
OK. I will next look into your calculation example with the light beams, and give my analysis.
 
Last edited:
Physics news on Phys.org
  • #107
DaleSpam said:
The clock analysis is the analysis of the clock itself, how its mechanism functions in any reference frame. It is, by your own admission, unaffected by how signals about its operation are transmitted to any observer, and so the analysis of those signals is unnecessary for the analysis of the clock mechanism, contrary to your premise.

My premise has nothing to do with the operation of the clock, it has to do with apparent effects. Like doppler and such things.

Why are we looking for T? You already agreed that it cannot affect the operation of the clock in any way. It is irrelevant.

Because T is the time shown in the unprimed frame when the observer receives the signal from B, which is different from the primed time, and which is the time that determines the perceived speed of the beam from the observer at the origin. It is the time the beam seems to take to cross the distance AB. It is far from irrelevant.

Besides the funny self-contradiction, it is one of the postulates of SR that it is going at c in all frames. We know its speed according to SR, and alternative theories that disagree with SR are not permitted here.

There's no contradiction whatsoever. Observed/detected light is going c in all frames. Einstein never once thought about how light would look like at a distance since it seemed ridiculous to do so, since we can't have any knowledge of undetected light. He and everyone else always addressed this issue by placing another observer in the path of light, but that says nothing about the observed behavior of light receding from the observer (I really believe that taking harrylin setup of scattering light from the beam's path plus my suggestion that we take this scattered light and bring it to one single point - the observer - is a novelty).
How do you measure the speed of the train? You take the distance it covers over the time it takes to cover it. This velocity is not the same as marked on the trains own speedometer, for instance. You are on the train and it marks .5c on the speedometer. You cross 1 light second, from your perspective, in 2 seconds.
An observer at the embankment would see the train cross that distance in 2 seconds + the time it takes light to reach back to him from that distance (which would be the time marked on the observers watch at the moment he sees the train at that distance). The speed will seem to be slower relative to the speed as seen on the onboard speedometer!

I have a diagram on this from a discussion I had a few months ago:
train_speed.jpg


But I'm afraid this will divert us from the topic, maybe this needs a thread itself.
 
  • #108
harrylin said:
Apparently he has corrected it now.

My previous analysis was correct, it just didn't include the extra light signal from the mirror back to the observer. Altergnostic apparently still thinks I made a mistake (which I didn't); I'll address his issue in a separate post.

harrylin said:
However, I missed why anyone would need a spatial rotation - S and S' are moving in parallel and the light ray reflects, it doesn't rotate.

You are correct that it doesn't matter how the x and y axes are oriented; either choice leads to the same physics. I switched it in my revised analysis only because I didn't see the point of getting into a protracted wrangle about it when I can do the analysis with altergnostic's preferred axis orientation and still show that there is no problem.
 
  • #109
harrylin said:
A bullet doesn't have the same speed in both frames; I'll stick to the topic instead, which allows much better clarity.

This is precisely my point! The same is true for the beam!

I already illustrated that it is not necessarily to use that light for the time data. So, if you mean that it is light or radio waves or other means that brings us the clock synchronisation, then I completely agree.

That's almost what I mean. I take that the sync has been done previously (at the origin) and the light actually brings time information, in this particular case. We have, at the origin, the same time marked both on the observer's watch as the light clock's time. The light clock departs. The observer will see the next second on the light clock at the moment he sees the signal, but the time marked on his watch when the signal reaches him is no t'B=1s, it is t'B + the time it takes light to cross BA.

In post #8 I gave you the answer (in the link). However, you did not comment on it.
[EDIT: In fact, my answer took care of the spatial rotation misconception]

I guess I forgot about that post. But there's no disagreement here.

Of course, we narrowed down the problem as stemming from a calculation error - either made by "all" textbooks and students, or made by you. My purpose was (and still is) to get rid of the bug, after which you can be an asset to this forum - and you will feel better. :smile:

You are welcome! That was ingenious indeed.

Thanks - such feedback is helpful to remain motivated with this voluntary work! :tongue2:

I started this post certain that the observer couldn't possibly trace the beam (and from current diagrams he really can't - you are the only one who made this possible so far in all the discussions I have participated, I really respect you for this). My last diagram was only possible because of this insight, I am only afraid that my conclusions are so far off current accepted diagrams that I won't get any credit for this. I don't care if I am wrong, really, I just want a really rigorous analysis, not just more of the same assumption that the beam must be traveling at the speed of light for this particular observer because it always travels at c for all observers - but I insist this is not your common observer, it is a non-observer, if you like. He is not observing the beam directly. The speed of light must apply to light that reaches him, since that is the light he detects going at c directly. The beam's velocity has to be calculated from that, just like you said above, the bullet has a different velocity in each frame, and so must this beam. I think this is even more consistent with such experiments than the opposite assumption that the beam must seem to move at c in this situation. Than we would have to explain why should the beam act differently from the bullet, since the setup is fundamentally the same.

OK. I will next look into your calculation example with the light beams, and give my analysis.

Thanks!
 
  • #110
altergnostic said:
My premise has nothing to do with the operation of the clock,
You have explicitly stated multiple times that the standard analysis of the operation of a light clock is incorrect or incomplete. Do you now agree that the standard analysis is both a correct and complete analysis of the operation of the clock? If not, then please state exactly your objection to the standard analysis of the operation of the clock which you agree has nothing to do with any signals sent to any observers.

Note, the standard analysis only purports to be an analysis of the operation of the clock and not an analysis of the subsequent transmission of the results to any observer. Such considerations are irrelevant to the operation, as you have already agreed.

altergnostic said:
Observed/detected light is going c in all frames. Einstein never once thought about how light would look like at a distance since it seemed ridiculous to do so, since we can't have any knowledge of undetected light.
You are factually wrong on this count. The postulate of SR is that all light pulses travel at c, regardless of whether they are detected or not. Please read the postulates and note that they do not mention anything about detection. The postulate is that light travels at c, end of story, no distinction whatsoever between light going towards or away from any observer or about the detectability of the light. Get your facts straight.
 
Last edited:
  • #111
PeterDonis said:
My previous analysis was correct, it just didn't include the extra light signal from the mirror back to the observer. Altergnostic apparently still thinks I made a mistake (which I didn't); I'll address his issue in a separate post.
You are correct that it doesn't matter how the x and y axes are oriented; either choice leads to the same physics. I switched it in my revised analysis only because I didn't see the point of getting into a protracted wrangle about it when I can do the analysis with altergnostic's preferred axis orientation and still show that there is no problem.

Sorry Peter, but I have to insist that you reconsider this carefully.
I insist that you have to align the x-axis with AB and completely ditch the speed of the light clock in the former x axis, since it is actually causing a distraction here.
Please note that the observer at the origin doesn't even need to know that speed to calculate anything:
He is given the distance AB
He knows the primed times, since he has synchronized his watch with the light clock back at the origin. He knows that the reflection at B occurs at t'=1s.
He receives the signal event from B at T=t'+ the time it took for the light signal to cross BA and reach him - that is the time he actually sees the signal, and is the time he observes the beam to reach the point B.

From the given distances and the observed times he can only observe that the beam crossed AB in T seconds, and then subtract the time it takes light to cross BA to get the primed time - which will be 1s again.

This seems all extremely consistent, and in accordance with what we would expect to have by replacing the beam with a projectile. If a projectile hits B at t'=2s, the observer will observe it crossing AB at T=t' + the time it took light to cross BA, and then subtract the time it takes light to cross BA to get the primed time - which will be 2s again.

By this method, the primed velocity will make the primed distance smaller than AB (length contraction), and the calculated distance will become y', which again is fully consistent with the setup. Like so:
Observed speed of the beam = AB/T = 1.12/2.12 =0.528c
Transformed time of event B = t'= 1s

We know that light travels at c locally from many many experiments, so to get the distance traveled by light in the primed frame, we just have to figure out the distance light crossed in 1 second (the primed time) = 1lightsecond = y'

Do you see my reasoning? You may not agree with it for some reason, but it is very consistent, and all the numbers add up. I can't see any error on it, if you can, please do point it out, since I am not here to defend any preconceived opinion, I'm just going where logic and math has taken me and sharing this with you for open analysis.

And just as a footnote, I don't think this contradicts any postulates nor disproves or changes SR in any way. It is simply a setup that hasn't been thought of yet, as far as I know (and I have to share credit for this with harrylin, even if he disagrees with my analysis). Observed light still travels at c. It also travels at c relative to the source. It is undetected light, or indirectly observed light, that I conclude doesn't need to follow the constant, but this is not your ordinary light. It is not the light that enters any known equation as c. This is light moving at a distance. Not a single light equation takes c to be light at a distance, everytime you have c in an equation, that c is referring strictly to light that reaches the observer, which is the light you do transforms with.
That's the importance of the signal arriving from B: it is with this light that we observe the beam, and it is with this light that we must do transforms, just like in any other setup.
To give the speed of light both for this incoming light and for the beam, as seen in the unprimed frame, is similar to believing that the speed of the man walking inside a moving train is the same for both primed (the train) and unprimed (embankment) frames.
If you still think the beam will be observed to go at c from A to B as observed from the observer at the origin A, than you have to explain why wouldn't the speed of a projectile (in place of the the beam) be the same in the primed and unprimed frame as well.

Please consider this carefully and think about this with no prejudice (I mean no harm to SR :smile:)
 
Last edited:
  • #112
altergnostic said:
Peter, you made what I believe to be the same mistake I pointed out earlier. You again plugged in the speed of the light clock into gamma.

It isn't a mistake (if there's any mistake involved here it's yours, in not understanding how relative motion is modeled in SR).

altergnostic said:
We are not looking for numbers for the light clock, we are looking for numbers for the beam.

Yes, but in order to get "numbers for the beam", you have to model the motion of the light clock as well. Otherwise you can't enforce the constraint that the beam has to be co-located with particular parts of the light clock at particular events (the source/detector at events D0 and D2, and the mirror at event B1a). Without that constraint in place any analysis of the beam's motion is meaningless.

altergnostic said:
My analysis is useless if you start assuming the beam going at c frim the beginning.

I didn't *assume* that the beam was moving at c; I *proved* that it moves at c, by enforcing the constraint I referred to above. Please read my analysis again, carefully. What I did was compute the locations of the light clock's source/detector at events D0 and D2, and compute the location of the mirror at event B1a, using only facts about the light clock's motion relative to the observer. I then showed that the spacetime interval between events D0 and B1a, and between events B1a and D2, is of zero length; this proves that the light pulse travels at c between those pairs of events. I did this in both frames to confirm that the interval is invariant (as it should be).

I also, once I understood that there was a second light signal involved (from event B1b to A2), did the same kind of computation for that light signal: compute the location of the observer at event A2 (we already know the mirror's location at event B1b because event B1b is co-located with event B1a), and confirm that the spacetime interval between events B1b and A2 is of zero length, again in both frames.

altergnostic said:
If you really want to humor me, place the x-axis aling with AB.

Sure, by all means let's do the analysis as many ways as you like; the answer will remain the same. :smile:

We now have the x-axis oriented in such a way that the event coordinates in the unprimed frame are as follows:

A0 = D0 = (0, 0, 0)

B1a = B1b = (t1, x1, 0)

A2 = (t2, 0, 0)

D2 = (t2, x2, y2)

Our objective is to find t1, x1, t2, x2, and y2, and then compute the spacetime intervals (A0 to B1a), (B1b to A2), and (B1b to D2), and verify that they are all zero.

The first thing to note is that the time coordinates are unchanged from my previous analysis; i.e., we still have t1 = 1.16 and t2 = 2.32.

The second thing to note is that x1 must be given by the sum of the squares of the x and y coordinates for events B1a/B1b in my previous analysis (since it's the same spatial distance, we've just rotated the axes to put that distance all along one axis); i.e., we must have

[tex]x_1 = \sqrt{ 1 + ( 0.58 )^2 } = 1.16[/tex]

The third thing to note is that the sum of the squares of x2 and y2 must equal the square of the y coordinate of event D2 in my previous analysis (again, the same spatial distance from the origin, just with the axes rotated); i.e., we must have

[tex]x_2^2 + y_2^2 = ( 1.16 )^2 = x_1^2[/tex]

This tells us something very important: the distances AB, BD, and AD in your diagram are all *equal*. (We could have seen this directly by noting that the light clock, which moves half as fast as the light pulse, covers distance AD in the same time as the light pulse covers distance AB + BD; therefore AB + BD = 2AD, which combined with AB = BD gives AB = BD = AD.) This means that the angle between lines AB and AD is 60 degrees (since it's an angle of an equilateral triangle), so we must have

[tex]\frac{y_2}{x_2} = tan (60 deg) = \sqrt{3}[/tex]

Substituting this into the equation above gives

[tex]x_2^2 + 3 x_2^2 = x_1^2[/tex]

or

[tex]2 x_2 = x_1[/tex]

Now we can confirm that the spacetime intervals I mentioned are zero. First, we have t1 = x1, so the interval (A0 to B1a) is obviously zero. Similarly, we have t2 - t1 = t1 = x1, so the interval (B1b to A2) is obviously zero. Finally, the interval (B1a to D2) is given by

[tex](t_2 - t_1)^2 - (x_2 - x_1)^2 - (y_2 - 0)^2 = ( 1.16 )^2 - ( x_2^2 + y_2^2 ) - x_1^2 + 2 x_2 x_1 = ( 1.16 )^2 - ( 1.16 )^2 - x_1^2 + x_1^2 = 0[/tex]

As I said, the answer remains the same. :smile:

altergnostic said:
Take the primed time for event B (1s) and the given distance AB (1.12).

These numbers are from different frames; trying to mix them in formulas is going to give you meaningless answers.

altergnostic said:
And the relative velocity Vab is the number we are seeking: the observed speed of the beam.

Oh, so by Vab you meant the speed of light, not the speed of the clock? Fine, but your calculation of it still makes no sense; once again, you can't mix numbers from different frames and expect to get meaningful answers. See above.
 
Last edited:
  • #113
altergnostic said:
I have a diagram on this from a discussion I had a few months ago

The second part of your diagram (the "TRAIN FRAME" part) is incorrect; in the train frame the blinker is motionless, so all of the space coordinates should be marked as zero.

altergnostic said:
But I'm afraid this will divert us from the topic, maybe this needs a thread itself.

Before starting one, I strongly advise you to review a good basic SR textbook, such as Taylor & Wheeler.
 
  • #114
PeterDonis said:
The second part of your diagram (the "TRAIN FRAME" part) is incorrect; in the train frame the blinker is motionless, so all of the space coordinates should be marked as zero.



Before starting one, I strongly advise you to review a good basic SR textbook, such as Taylor & Wheeler.

That is the train frame measuring its velocity against the ground, just like you do in your car. It is the speed marked on its apeedometer.
 
  • #115
altergnostic said:
Sorry Peter, but I have to insist that you reconsider this carefully.
I insist that you have to align the x-axis with AB and completely ditch the speed of the light clock in the former x axis, since it is actually causing a distraction here.

Done; see my previous post (actually the last but one from this one). The answer remains the same.

altergnostic said:
He is given the distance AB
He knows the primed times, since he has synchronized his watch with the light clock back at the origin.

But he can't use the primed times along with unprimed distances to calculate anything meaningful. To correctly calculate a speed you must use the distance and time from the same frame. The "distance AB" that he is given is in the unprimed frame. That distance is *not* perpendicular to the relative motion of the light clock and the observer in the unprimed frame (it is in the *primed* frame, but it is *not* in the unprimed frame--perhaps this is one of the points where you are confused). So the observer *can't* combine it with a time in the primed frame to get a meaningful answer, because of length contraction.

altergnostic said:
From the given distances and the observed times he can only observe that the beam crossed AB in T seconds, and then subtract the time it takes light to cross BA to get the primed time - which will be 1s again.

Nope, this mixes numbers from different frames again.

altergnostic said:
This seems all extremely consistent, and in accordance with what we would expect to have by replacing the beam with a projectile.

The velocity of an object moving at less than c *does* change when you change frames. The velocity of light does not. Please review a good basic relativity textbook, paying particular attention to the formula for relativistic velocity addition; you will note that that formula always gives c for the velocity of a light beam.

altergnostic said:
Do you see my reasoning? You may not agree with it for some reason, but it is very consistent, and all the numbers add up. I can't see any error on it, if you can, please do point it out, since I am not here to defend any preconceived opinion, I'm just going where logic and math has taken me and sharing this with you for open analysis.

Read my latest analysis again, carefully; also read my comments above, carefully. Your error is that you are mixing numbers from different frames and expecting them to give you meaningful answers. Also, you may be mistakenly assuming that the distance AB is the same in both frames; it's not, for the reason I gave above.

altergnostic said:
And just as a footnote, I don't think this contradicts any postulates nor disproves or changes SR in any way.

Then you don't understand SR very well.

altergnostic said:
Observed light still travels at c. It also travels at c relative to the source. It is undetected light, or indirectly observed light, that I conclude doesn't need to follow the constant, but this is not your ordinary light.

[rest of post snipped]

This is all confused. All light travels at c in SR, in any inertial frame. If you read my latest analysis, you will see that the light beam traveling from B1a to D2 travels at c, even though it is not "observed" (both events are "at a distance" from the observer).
 
  • #116
altergnostic said:
That is the train frame measuring its velocity against the ground, just like you do in your car. It is the speed marked on its apeedometer.

But the ground is moving in the opposite direction, and the blinker is not moving with it. A correct "TRAIN FRAME" diagram would have the blinker on the right, motionless, and points on ground moving to the left. Then you would have to calculate *which* points on the ground would be at which spatial coordinates in the train frame at which times.

A better tool for this job, IMO, would be a proper spacetime diagram, since that directly shows both space and time relationships and allows you to draw actual worldlines so you can see how they are related. If you aren't familiar with spacetime diagrams, I would recommend learning them; they really help to remove confusion for scenarios like this one.
 
  • #117
PeterDonis said:
But the ground is moving in the opposite direction, and the blinker is not moving with it. A correct "TRAIN FRAME" diagram would have the blinker on the right, motionless, and points on ground moving to the left. Then you would have to calculate *which* points on the ground would be at which spatial coordinates in the train frame at which times.

A better tool for this job, IMO, would be a proper spacetime diagram, since that directly shows both space and time relationships and allows you to draw actual worldlines so you can see how they are related. If you aren't familiar with spacetime diagrams, I would recommend learning them; they really help to remove confusion for scenarios like this one.

Agreed. But still it is not hard to see, from the bottom diagram, that the speed of the ground measured from the train (the speed on its speedometer, to simplify things) will not be the same speed as that observed by the man at the origin. This actually compromises gamma a bit, since it is hard to decide which velocity to plug into it.

But please, let's leave this be for now, since we will surely divert from the topic. If you feel this deserves attention and want to discuss it further, I will gladly start a thread on this, but let's please not digress here, since this thread is already dense enough as it is. Deal?
 
  • #118
altergnostic said:
This is precisely my point! The same is true for the beam!
No, it isn't. The invariance of the speed of light is a postulate of SR. If that is precisely your point then your point is WRONG.
 
  • #119
altergnostic said:
Agreed. But still it is not hard to see, from the bottom diagram, that the speed of the ground measured from the train (the speed on its speedometer, to simplify things) will not be the same speed as that observed by the man at the origin.

You're going to have to explain how this is "not hard to see", because I don't see it. The only thing that changes about the velocity from frame to frame is the sign: in the ground frame, the train moves at +v, and in the train frame, the ground moves at -v.

I suspect you are confused because you think length contraction affects the velocity from frame to frame. It doesn't, because length contraction and time dilation *both* come into play, and their effects cancel when applied to the relative velocity. If you don't see how that works, read through my analysis again, carefully; for example, look at how the coordinates of events A2 and D2 transform between the primed and the unprimed frame, and note that the time and space coordinates change in concert to keep the relative velocity the same.

[Edit: To expand on this a little more, events D0 and D2 both lie on the worldline of the light clock source/detector, which is equivalent to the "train"; and events A0 and A2 both lie on the worldline of the observer, which is equivalent to the "ground". Events A0 and D0 are co-located at the origin, so their coordinates drop out of the analysis, and we can use the coordinates of event D2 in the unprimed frame, and A2 in the primed frame, to compute the relative velocity. In the unprimed frame, event D2 has coordinates (t2, x2, y2), and the relative velocity is given by sqrt(x2^2 + y2^2) / t2, which works out to 0.5. In the primed frame, event A2 has coordinates (t2', x2', y2'), and the relative velocity is given by sqrt(x'2^2 + y2'^2) / t2', which works out to 0.5. Then we just have to remember to set the sign appropriately, based on the sign of (x2, y2) or (x2', y2'), which gives v = +0.5 in the unprimed frame and v = -0.5 in the primed frame.]

altergnostic said:
But please, let's leave this be for now, since we will surely divert from the topic. If you feel this deserves attention and want to discuss it further, I will gladly start a thread on this, but let's please not digress here, since this thread is already dense enough as it is. Deal?

I don't see how starting another thread will help. This is an absolutely fundamental point that underlies the "problem" you posed in the OP, and all of my analysis, and indeed all of SR. If you think it's wrong, you basically think SR is wrong; and if you can't back your claim up cogently (which you haven't so far) and you won't abandon it, any further discussion of your claim is basically out of bounds here on PF, since we don't discuss personal speculations that contradict SR. Moving the discussion to another thread won't change that.
 
Last edited:
  • #120
On the left, the drawing above the Ux axis is a space-time description of the 1-dimensional motion of observer A while monitoring his light clock.
The events (small circles) involve 2-dimensional motion of light in an arbitrary plane perpendicular to the Ux axis, labelled as Up. The Ut axis is only labelled in terms of t and gamma for the general case. Applying the drawing to the specific case v=.5c, gamma =1.155, d=1 ls, and t=1. Coordinate notation is (x,p,t).
Event sequence is:
U1(0,0,0) emit photon,
U2(.578,1.00,.578) reflect,
U3(1.16,0,1.16) detect.
On the right A's motion has been removed, and time dilation (.866) applied.
Event sequence is:
A1(0,0,0)
A2(0,1,1)
A3(0,0,2)
A is coincident with events 1 and 3, therefore values for event 2 are calculated based on additional information (symmetry and constant c).
U is coincident with event 1, therefore values for events 2 and 3 are calculated based on additional information (some form of light interaction between devices or observer A).

forum light clock.gif
 
  • #121
PeterDonis said:
But he can't use the primed times along with unprimed distances to calculate anything meaningful. To correctly calculate a speed you must use the distance and time from the same frame. The "distance AB" that he is given is in the unprimed frame. That distance is *not* perpendicular to the relative motion of the light clock and the observer in the unprimed frame (it is in the *primed* frame, but it is *not* in the unprimed frame--perhaps this is one of the points where you are confused). So the observer *can't* combine it with a time in the primed frame to get a meaningful answer, because of length contraction.

That's right! But you are confusing the coordinates. He uses the primed distances and unprimed times. And that is allowed because he is directly seeing the km marks, you see. I have determined that in the setup, all distances have been previously walked and each km marked with a big sign. The observer can see those directly. The primed distances are given, and the speed of the beam is not. Without knowing the speed of the beam nor the distances, we would be short on variables. Since I am looking for the velocity of the beam, I have given the distances.

Nope, this mixes numbers from different frames again.
But we have to! Just like in a projectile analysis, the velocities are different for each frame. If I am correct and this is the case here too, we have to be given some set of coordinates. If you think it is necessary (and I don't, but I may be wrong) you can transform the distance AB into unprimed distances and do your analysis. Still you should find different observed velocities for the beam.


The velocity of an object moving at less than c *does* change when you change frames. The velocity of light does not. Please review a good basic relativity textbook, paying particular attention to the formula for relativistic velocity addition; you will note that that formula always gives c for the velocity of a light beam.

I am very aware of this, and this is where the problem lies. It is an assumption, I repeat. This beam cannot go at c on both frames just as a projectile on that same path can't have the same velocity in both frames. If you disagree, please explain HOW one setup differs from the other, and why should the beam's apparent velocity not change just like the projectile's. Remember, this beam is not seen directly, you have to find its velocity from the signals. This is a premise from the setup - how does the observer at A determines the speed of the beam given primed distances, primed times and observed (unprimed) times?


Read my latest analysis again, carefully; also read my comments above, carefully. Your error is that you are mixing numbers from different frames and expecting them to give you meaningful answers. Also, you may be mistakenly assuming that the distance AB is the same in both frames; it's not, for the reason I gave above.

Done. The distances are not the same for both frames, but the observer at A doesn't have a number for the (observed) velocity to determine the distance, he only has the observed time and the given distance (and also the primed time). Also, he has direct visual information of the primed distances, so he doesn't even need to transform h into h' - it is a given. That's enough variables to solve.


Then you don't understand SR very well.
This is all confused. All light travels at c in SR, in any inertial frame. If you read my latest analysis, you will see that the light beam traveling from B1a to D2 travels at c, even though it is not "observed" (both events are "at a distance" from the observer).

Notice we only need to focus on AB to reach our conclusions, since the question in simply if the speed of the beam is the same for both frames. You can end your analysis before taking D into consideration.

But you only get c for the beam because you are inserting the speed of the light clock into gamma, and you are not calculating the speed of the beam from the signals at all. Forget the light clock's velocity. Take it that the observer has no velocity information whatsoever. He has only primed and unprimed times and unprimed distances.
What you are actually finding is the speed of the beam relative to the light clock! Ditch the light clock's velocity, assume the observer has no acces to it. That velocity was there only to determine the hypotenuse, nothing more, but that can be a given (h').

Just as in standard light clock diagrams, you don't even need to use gamma - the time dilation transforms can actually derived from these diagrams. But they can only do it just like you did it: by inserting the speed of the light clock and with no consideration of how can the observer possibly determine the path of the beam, since it is not really observed.

The only data available to the observer is:
t'A = tA = 0s
t'B = 1s
AB = h' = 1.12 lightseconds
AC = y'= 1 lightsecond

From this, we conclude that the observed time for the event at B is simply:
tB = t'B + the time it takes for the signal from B to reach A (which is the time it takes light to cross AB).

Now find the speed of the beam as seen from A.

Conversely, replace the beam with a projectile, with this set of givens:
t'A = tA = 0s
t'B = 2s (this is the moment the projectile impacts B as seen from the primed frame)
AB = h' = 1.12 lightseconds
AC = y'= 1 lightsecond

Now find the speed of the projectile as seen from A.

The setup is the same and the operations should be equivalent. If both operations are not equivalent, tell my why. If the reason is something like "because the beam can't vary its speed" then you are assuming what you are trying to prove.

I can't settle with your analysis while you are using the speed of the light clock in the equations. You have to find the speed of the beam from the given times and distances alone, than we will have analysed the problem with the same set of givens and see in what ways we disagree.
 
  • #122
DaleSpam said:
No, it isn't. The invariance of the speed of light is a postulate of SR. If that is precisely your point then your point is WRONG.

Then prove it! Find the speed of the beam with my set of givens. The speed of the beam can be directly calculated with the givens.

I state again that the speed of light is c for directly observed light. It is c independent of the motion of the source or the receiver. But that says nothing about a non-receiver. The light signals arriving at A are traveling at c, as the postulate demands, but since the observer can't see the beam directly, he MUST calculate its speed based on signals from reflection events (in this setup).

I want you to calculate the speed of the beam. It is an inverse operation, to check if the beam is really observed to move at c. The givens are clear:

t'A = tA = 0s
t'B = 1s
h'= 1.12 lightseconds

you can find the time the observer sees the signal from B like so:
tB = t'B + the time it takes the signal from B to reach A

Calculate the speed of the beam.

For good measure, replace the beam with a projectile, with the givens:
t'A = tA = 0s
t'B = 2s
h'= 1.12 lightseconds

Find the speed of the projectile.

If the operations are not equivalent, please explain me why. If the reason is something like "because the beam must travel at c" than you are assuming what the setup tries to prove.
 
  • #123
PeterDonis said:
You're going to have to explain how this is "not hard to see", because I don't see it. The only thing that changes about the velocity from frame to frame is the sign: in the ground frame, the train moves at +v, and in the train frame, the ground moves at -v.

I suspect you are confused because you think length contraction affects the velocity from frame to frame. It doesn't, because length contraction and time dilation *both* come into play, and their effects cancel when applied to the relative velocity. If you don't see how that works, read through my analysis again, carefully; for example, look at how the coordinates of events A2 and D2 transform between the primed and the unprimed frame, and note that the time and space coordinates change in concert to keep the relative velocity the same.

[Edit: To expand on this a little more, events D0 and D2 both lie on the worldline of the light clock source/detector, which is equivalent to the "train"; and events A0 and A2 both lie on the worldline of the observer, which is equivalent to the "ground". Events A0 and D0 are co-located at the origin, so their coordinates drop out of the analysis, and we can use the coordinates of event D2 in the unprimed frame, and A2 in the primed frame, to compute the relative velocity. In the unprimed frame, event D2 has coordinates (t2, x2, y2), and the relative velocity is given by sqrt(x2^2 + y2^2) / t2, which works out to 0.5. In the primed frame, event A2 has coordinates (t2', x2', y2'), and the relative velocity is given by sqrt(x'2^2 + y2'^2) / t2', which works out to 0.5. Then we just have to remember to set the sign appropriately, based on the sign of (x2, y2) or (x2', y2'), which gives v = +0.5 in the unprimed frame and v = -0.5 in the primed frame.]



I don't see how starting another thread will help. This is an absolutely fundamental point that underlies the "problem" you posed in the OP, and all of my analysis, and indeed all of SR. If you think it's wrong, you basically think SR is wrong; and if you can't back your claim up cogently (which you haven't so far) and you won't abandon it, any further discussion of your claim is basically out of bounds here on PF, since we don't discuss personal speculations that contradict SR. Moving the discussion to another thread won't change that.

Well... I'm sorry to hear that. But I'll try again, anyway, since this is not a personal opinion, it is a direct mathematical and logical operation:

A train moves along the tracks with km markers on it, placed at every lightsecond distance. The train moves along the tracks at 0.5c, as seen on an onboard speedometer (conversely, the train observers the lightsecond markers moving at 0.5c, which amounts to the same thing).
As each lightsecond sign passes the train by, the train emits a signal to the observer at the origin. This signal acts like a clock, blinking once every 2 lightseconds.
At the origin, the observer's watch has been synchronized with the blinker, ticking once every two seconds.

By the time the train reaches the first lightsecond sign, it marks 2s and blinks. The signal has to cross the lightsecond distance back to the origin, which takes one second, so it reaches the origin 2 seconds after the blink event, so the observer will see the train (the blink) at the first lightsecond mark when his watch marks 3 seconds.

By direct observation, he will see the train moving at 1/3c, in disagreement with the train's onboard speedometer. The velocities are different for each frame.

The velocities are symmetric only if you transform from the velocity of the train as seen from the embankment to the velocity of the embankment as seen from the train, which is not what we are looking for. We are comparing the speed of the train relative to the tracks as observed from the embankment to the speed of the train relativ to the tracks as observed from the train.
 
  • #124
altergnostic said:
Then prove it!
Sure: http://www.edu-observatory.org/physics-faq/Relativity/SR/experiments.html

altergnostic said:
Find the speed of the beam with my set of givens.
The speed of any beam of light is c. Your givens may not be self-consistent or consistent with relativity so contradictory calculations from your givens don't prove anything. [EDIT: I checked and your "givens" are in fact wrong. My calculations agree with PeterDonis except for some rounding differences where I got e.g. 1.15 and he got 1.16. See details below]
 
Last edited:
  • #125
harrylin said:
[..] A bullet doesn't have the same speed in both frames; I'll stick to the topic instead, which allows much better clarity. [..]
altergnostic said:
This is precisely my point! The same is true for the beam! [..] the bullet has a different velocity in each frame, and so must this beam. [..]
That is the exact contrary of the premises of SR. So, you are stuck at the starting point, as you still think that the following are irreconcilable :
We will raise this conjecture (the purport of which will hereafter be called the “Principle of Relativity”) to the status of a postulate, and also introduce another postulate, which is only apparently irreconcilable with the former, namely, that light is always propagated in empty space with a definite velocity c which is independent of the state of motion of the emitting body.
- http://www.fourmilab.ch/etexts/einstein/specrel/www/

Now the calculation, which isn't anything really - it only gets interesting when you consider mutual measurements, but you didn't yet reach that point.
Anyway:
altergnostic said:
[..]
Also (from earlier):
"Just notice that by taking notes of the times of each x,y position, and the length of the line imprinted on the plate, you will directly calculate light to travel faster than c."

Perhaps it's time for you to present a calculation, based on either that figure or the photographic plates, and which you think would produce a speed of light greater than c. Probably an interpretation error will show up (but perhaps a calculation error).
[..] The setup should have the observer fixed at some origin. Consider this:

lightclock_OK.jpg


All lengths and distances are primed because they are all locally measured numbers. If you like, imagine that before the experiment we have walked all the distances and measured them with a measuring rod. You can even imagine that we have marked each km with a big sign with the km distance painted on it, so even in the moving system S we know the primed spatial coordinates from direct observation.

Fix an obsever at A, his frame denoted by S.

V=c/2= the velocity of the light clock in the x direction, as shown on the onboard speedometer. Also, this velocity has been previously agreed upon, so there's no doubt that's the speed of the light clock.

BC=y'=1 lightsecond = the distance between mirrors (measured locally, remember) and this is a given.
It's a bit unusual to call the moving frame S', but never mind, Still, I will modify your symbols a little: it's much less confusing if we give everything that is measured with S' an ' and no ' for everything that is measured with S. Thus:

BC=y=1 lightsecond = the distance between mirrors (measured with S) and this is a given.
In addition, according to the second postulate (see above) the same must be true as measured with S'. There is no disagreement about the height of the train: y=y'= 1 lightsecond according to both S and S'. I hope that you see that. A light ray that is sent from C to B will take 1 s according to S'.

AC=x't'=0.5 lightseconds = distance traveled by the light clock in the x direction in 1 s.

TA=T'A= 0 = time of first recorded reflection, at both S and S' origins, which are both at A.

T'B≠1s. It is the time of the second reflection event as calculated in the rest frame S', based on the fact that CB= 1 light second. As a matter of fact, if according to S', two light flashes are sent simultaneously from A and C, the flash from C will arrive after 1s but the path length AB is longer, so it must take longer. Of course, it is assumed that the speed of light is the same in all directions; else no prediction is possible at all.

The second postulate effectively says that you may set up an inertial reference system, pretend that it is at rest in the ether, and everything will work as if your assumption was correct - just as in classical mechanics one may pretend that the system is "truly" in rest. And the error that you made here is literally a "classic": the same error was made by Michelson in 1886. As he admitted in 1887:
In deducing the formula for the quantity to be measured, the effect of the motion of the Earth through the ether on the path of the ray at right angles to this motion was overlooked.
- https://en.wikisource.org/wiki/On_the_Relative_Motion_of_the_Earth_and_the_Luminiferous_Ether

I think that you should read that, or look at an animation such as here:
http://galileoandeinstein.physics.virginia.edu/more_stuff/flashlets/mmexpt6.htm
(note: ignore the word in red - hopefully that bug will be fixed in the coming week!)

Anyway, let's continue the -now trivial- analysis:

T'B2=T'B + T'BA = the moment a reflection signal (i.e.: scatter event) from B is seen at A in S, which is the time light takes to travel from the reflection event T'B to the observer at A. Of course, the trajectories AB and BA are the same, along h', and are assumed to take the same time.

This triangle represents the first two reflection events as seen in the moving frame S.

The light clock is going at 0.5c, as given by an onboard speedometer, useful in this setup to determine AC. [EDIT: correct one more "given" error: also AC was wrong!]
After 1 s in S' the light from A has not yet arrived at the top mirror and the train moves beyond x=0.5; xC=0.5h'. The angle is 60° as you can easily verify. [STRIKE]Pythagoras[/STRIKE]Trigonometry: Light path in S' AB = h'=1/sin(60)= 1.155.
Or, still with Pythagoras: 12-0.52=1/k2 =>k=1.155
h'=k*1=1.155 lightseconds.
Thus one mirror scatter is seen at A when T'=T=0 and the other at B when T=1, and T'=1.155

Note that you did not draw the figure for S, which is simply vertical with y=y'.

AB = h' = 1.155
T'B2 = 2T'B = 2.31 s
VAB = 2h'/T'B2 = 1 lightsecond/s

Does that help? Note that this is just the beginning...

In fact, it is quite useless to proceed if you don't master for example a classical analysis of the Michelson-Morley experiment.
 
Last edited:
  • #126
altergnostic said:
Well... I'm sorry to hear that. But I'll try again, anyway, since this is not a personal opinion, it is a direct mathematical and logical operation:

A train moves along the tracks with km markers on it, placed at every lightsecond distance. The train moves along the tracks at 0.5c, as seen on an onboard speedometer (conversely, the train observers the lightsecond markers moving at 0.5c, which amounts to the same thing).

If I'm understanding you correctly, you've neglected to include the effects of length contraction. If the light second markers are placed every 1 second apart in the rest frame of the tracks, in the moving frame of the train they'll be closer together.

This is why "celerity" akak "prover velocity" is a different concept than velocity in SR.

See for instance http://arxiv.org/abs/physics/0608040 by Thomas Greber.

Intergalactic spaceflight: an uncommon way to relativistic kinematics and dynamics

In the Special Theory of Relativity space and time intervals are different in different frames of reference. As a consequence, the quantity 'velocity' of classical mechanics splits into different quantities in Special Relativity, coordinate velocity, proper velocity and rapidity. The introduction and clear distinction of these quantities provides a basis to introduce the kinematics of uniform and accelerated motion in an elementary and intuitive way.

...

Sometimes, one single quantity in a theory splits into several distinct quantities in the generalised theory. One example is the velocity in classical mechanics. In the widest sense, velocity or speed means the covered distance divided by the time needed to cover it. This is uncritical in classical physics, where time and distance are well defined operational concepts that are independent of the frame of reference, in which they are measured. In Special Relativity, these concepts depend on the frame of reference in which they are defined, if the frames are not at rest with respect to each other. This makes it necessary to distinguish between the different possibilities regarding the frame of reference in which the spatial and the temporal intervals are measured. This is best illustrated with a common situation of measuring the velocity of a rolling car. Firstly, the velocity of the car can be measured by driving past kilometer posts and reading the time at the moment of passing the post on synchronized watches mounted on the posts. Secondly, the driver can also measure the velocity by reading the corresponding times on a clock which is traveling with the car. Thirdly, a person with clock standing beside the street can measure the times on his clock at the moments, when the front and the rear ends of the car are passing him. The traveled distance is then taken from a measurement of the length of the car in the frame of reference of the car. A fourth possibility measures the velocity of the car up to an arbitrary constant by measuring its acceleration using an accelerometer traveling with the car, e.g. by measuring a force and using Newtons Second Law, and integrates the measured acceleration over the time measured with a clock, also traveling with the car.

In classical mechanics, all four measurements are equivalent and give the same value for the velocity. In Special Relativity, the first possiblility gives the coordinate velocity, which is often referred to as the genuine velocity. The second and third possibilities are equivalent, but are hybrid definitions of the speed. The temporal and spatial intervals are measured in different frames of reference. This speed is sometimes called celerity [1, 2], or proper velocity [3, 4]. In addition, proper velocity is the spatial part of the vector of four-velocity [5]. The fourth definition of a speed, sometimes called rapidity [1, 2], is somewhat distinct from the other concepts of speed in so far as it can only be determined as a change of speed. The need to measure an acceleration in the moving frame by means of measuring a force qualifies rapidity to bridge kinematics and dynamics. This seems to be not critical in classical mechanics, if the concept of force is accepted as an operational quantity. However, it can also be used to determine the relativistic version of Newtons Second Law if viewed from the accelerated frame of reference.
 
  • #127
altergnostic said:
That's right! But you are confusing the coordinates. He uses the primed distances and unprimed times. And that is allowed because he is directly seeing the km marks, you see. I have determined that in the setup, all distances have been previously walked and each km marked with a big sign. The observer can see those directly.

No, he can't. He has to get light signals from them, just like from everything else that's not at his spatial location. Putting km markers everywhere doesn't magically transmit information instantaneously.

In any case, no matter how he gets the numbers, you can't mix numbers from different frames and expect to get meaningful answers. Any valid calculation of the relative velocity has to use either an unprimed distance and corresponding unprimed time, or a primed distance and a corresponding primed time. Using a primed distance and unprimed time is just as meaningless as using an unprimed distance and a primed time.

altergnostic said:
But we have to! Just like in a projectile analysis, the velocities are different for each frame.

For an object that moves at less than the speed of light, the velocity of that same object is different in different frames, yes. But that doesn't apply to light; the velocity of a light beam is c in every frame (assuming inertial frames and flat spacetime). I've explicitly proven that to be the case in your scenarios.

Also, you have claimed that the *relative* velocity of two frames is different in one frame than in the other; this is the basis of your claim that you don't know which gamma factor to use. The fact that the velocity of the same object (if it moves slower than light) is different in different frames is irrelevant to that claim, because here you are not comparing the velocity of the same object in different frames; you are comparing the velocities of *different* objects in different frames. You are comparing the velocity of the train in the observer frame, with the velocity of the observer in the train frame. Those two velocities have the same magnitude, but opposite directions; I've proven that.

altergnostic said:
I am very aware of this, and this is where the problem lies. It is an assumption, I repeat.

And I repeat, this is *not* an assumption; it is a proven fact. I've proven it in my previous posts. You can prove that all light beams travel at c using only facts about the relative motion of the light clock and the observer (or the train and the ground), without making any assumptions about the speed of the light beam.

altergnostic said:
This beam cannot go at c on both frames just as a projectile on that same path can't have the same velocity in both frames. If you disagree, please explain HOW one setup differs from the other, and why should the beam's apparent velocity not change just like the projectile's.

Because the beam is moving at c, and the projectile is moving slower than c. The two cases are different. That's just a fact about how relativistic spacetime works. The only assumption I can see that is worth mentioning here is the isotropy of space (since that's really what underlies the claim that the velocity of the observer relative to the clock is equal in magnitude and opposite in direction to the velocity of the clock relative to the observer). That assumption is supported by a massive amount of evidence, so I don't see the point of questioning it here.

altergnostic said:
Remember, this beam is not seen directly, you have to find its velocity from the signals. This is a premise from the setup - how does the observer at A determines the speed of the beam given primed distances, primed times and observed (unprimed) times?

By using known facts about the speed of the light clock relative to him to establish the coordinates of the relevant events in the unprimed frame, independently of the speed of the light signals that travel between those events.

altergnostic said:
But you only get c for the beam because you are inserting the speed of the light clock into gamma

Well, of course. Why not?

altergnostic said:
and you are not calculating the speed of the beam from the signals at all.

I don't know what "calculating the speed of the beam from the signals" means. I'm calculating the speed of the beam from knowledge of the distance and time intervals between two known events on the beam's worldline.

altergnostic said:
Forget the light clock's velocity. Take it that the observer has no velocity information whatsoever. He has only primed and unprimed times and unprimed distances.

Then I can re-write all the formulas in terms of the unknown gamma factor for the relative velocity, and I will *still* get the same answer. All the factors of gamma end up canceling out. You should be able to see that from my analysis, but if not, I can re-write it this way (although you really should be able to do that yourself). In fact, I actually do re-write the formula for the velocity of the light beam in the light clock this way (for comparison with the velocity of a projectile in a "projectile clock") below.

altergnostic said:
The only data available to the observer is:
t'A = tA = 0s

Yes.

altergnostic said:
t'B = 1s

Yes. But you are aware, aren't you, that this implies tB = gamma, right?

altergnostic said:
AB = h' = 1.12 lightseconds

Is this supposed to be a distance in the unprimed frame? If so, how do you know it is consistent with the values for tA, t'A, and t'B that you just gave? If you start out with an inconsistent set of premises, of course you're going to get a meaningless answer.

altergnostic said:
AC = y'= 1 lightsecond

I don't see how this follows at all. AC is supposed to be the distance, in the unprimed frame, that the light clock source/detector travels during the time of flight of the beam, correct? In that case, it should be gamma, not gamma primed, because the time of flight of the beam, in the unprimed frame, is tB = gamma (see above).

So already I have spotted two wrong premises in your argument; no wonder you're getting erroneous answers.

altergnostic said:
Conversely, replace the beam with a projectile, with this set of givens:
t'A = tA = 0s

Yes.

altergnostic said:
t'B = 2s (this is the moment the projectile impacts B as seen from the primed frame)

Yes. But again, you are aware that this implies tB = 2 gamma, right?

altergnostic said:
AB = h' = 1.12 lightseconds
AC = y'= 1 lightsecond

Same comment as above. This value of AB is not consistent with the above, and AC is 2 gamma, not gamma primed, or even 2 gamma primed, which is what you should have written by the same reasoning as the above--t'B is twice as large, so AB should be twice as large as well since the light clock spends twice as much time traveling as before. You are aware, aren't you, that using a projectile traveling at 0.5c, instead of a light beam traveling at c, changes the geometry of your triangle diagram, right? It's *not* the same triangle.

altergnostic said:
The setup is the same and the operations should be equivalent. If both operations are not equivalent, tell my why. If the reason is something like "because the beam can't vary its speed" then you are assuming what you are trying to prove.

I've already done the light beam case in my previous analysis, but perhaps it would help to do a similar analysis on the "projectile clock" case; we'll assume a projectile traveling at 0.5c in the primed frame, in the direction perpendicular to the relative motion of the light clock and the observer in that frame. I'll orient the x-axis in the direction the projectile travels, and the y-axis in the direction of the relative motion of the "projectile clock" and the observer. (Yes, I know you think that the x-axis has to point directly along the path to where the mirror will be in the unprimed frame when the projectile hits it. I've already shown that that doesn't matter; I don't think it needs to be shown again. But rotating the spatial axes will be easy enough after the analysis I'm about to show, if you really insist on seeing it done that way.)

We are given the following event coordinates:

[tex]A0 = D0 = A0' = D0' = (0, 0, 0)[/tex]
[tex]B1a' = B1b' = (2, 1, 0)[/tex]
[tex]D2' = (4, 0, 0)[/tex]

This assumes a "projectile" traveling at 0.5c in the primed frame, in the x-direction.

The transformation equations are (I'm writing them now in terms of an unknown gamma factor, since you are now saying we don't know the relative velocity of the light clock and the observer):

Unprimed to Primed:

[tex]t' = \gamma ( t - v y )[/tex]
[tex]x' = x[/tex]
[tex]y' = \gamma ( y - v t )[/tex]

Primed to Unprimed:

[tex]t = \gamma ( t' + v y' )[/tex]
[tex]x = x'[/tex]
[tex]y = \gamma ( y' + v t' )[/tex]

This yields the following coordinates for events in the unprimed frame:

[tex]B1a = B1b = ( 2 \gamma, 1, 2 \gamma v )[/tex]
[tex]D2 = ( 4 \gamma, 0, 4 \gamma v )[/tex]

The velocity of the projectile in the unprimed frame on each leg can then be computed like this:

[tex]v_{projectile} = \frac{\sqrt{\Delta x^2 + \Delta y^2}}{\Delta t} = \frac{\sqrt{1 + 4 \gamma^2 v^2}}{2 \gamma} = \frac{1}{2} \sqrt{(1 - v^2) \left( 1 + \frac{4 v^2}{1 - v^2} \right)} = \frac{1}{2} \sqrt{\frac{(1 - v^2)(1 - v^2 + 4 v^2)}{1 - v^2}} = \frac{1}{2} \sqrt{1 + 3 v^2}[/tex]

This will be somewhere between 1/2, which is the velocity of the projectile in the primed frame, and 1 (but always less than 1 for v < 1); but it is only *equal* to 1/2 for v = 0, so you are correct that the velocity of the projectile in a "projectile clock" *does* change if you change frames.

I could leave it to you to spot the difference between this and the case of the light clock, but I suppose I'll go ahead and give it; the corresponding formula from my previous analysis would be (written with an unknown gamma factor to make it clear how it drops out of the analysis):

[tex]v_{beam} = \frac{\sqrt{\Delta x^2 + \Delta y^2}}{\Delta t} = \frac{\sqrt{1 + \gamma^2 v^2}}{\gamma} = \sqrt{(1 - v^2) \left( 1 + \frac{v^2}{1 - v^2} \right)} = \sqrt{\frac{(1 - v^2)(1 - v^2 + v^2)}{1 - v^2}} = 1[/tex]

As you can see, this formula always gives 1, regardless of v (as long as v < 1). So unlike the projectile case, the speed of a light beam in a light clock does *not* change when you change frames.
 
  • #128
Just to put some icing on the cake, here's an even more general formula which works for arbitrary "projectile" velocities (thus including the case of the light beam as the "projectile" as well).

We have a clock using a "projectile" that travels between a source/detector and a reflector. The projectile's velocity is [itex]v_p'[/itex] in the rest frame of the clock. We are given the following event coordinates (the projectile travels in the x direction):

[tex]A0 = D0 = A0' = D0' = (0, 0, 0)[/tex]
[tex]B1a' = B1b' = (1 / v_p', 1, 0)[/tex]
[tex]D2' = (2 / v_p', 0, 0)[/tex]

The transformation equations are as before, and they yield the following coordinates for events in the unprimed frame:

[tex]B1a = B1b = ( \gamma / v_p', 1, \gamma v / v_p')[/tex]
[tex]D2 = ( 2 \gamma / v_p', 0, 2 \gamma v / v_p')[/tex]

The velocity of the projectile in the unprimed frame on each leg is then:

[tex]v_p = \frac{\sqrt{\Delta x^2 + \Delta y^2}}{\Delta t} = \frac{\sqrt{1 + \gamma^2 v^2 / v_p'^2}}{\gamma / v_p' } = v_p' \sqrt{(1 - v^2) \left( 1 + \frac{v^2}{v_p'^2 (1 - v^2 )} \right)} = v_p' \sqrt{\frac{(1 - v^2)(1 - v^2 + v^2 / v_p'^2)}{1 - v^2}} = v_p' \sqrt{1 + v^2 \left( \frac{1}{v_p'^2} - 1 \right)}[/tex]

For [itex]0 < v_p' < 1[/itex], this gives [itex]v_p' < v_p < 1[/itex] for any [itex]0 < v < 1[/itex]. For [itex]v_p' = 1[/itex], this gives [itex]v_p = 1[/itex] for any [itex]0 < v < 1[/itex]. QED.
 
  • #129
altergnostic said:
The givens are clear:

t'A = tA = 0s
t'B = 1s
h'= 1.12 lightseconds
Your givens are wrong. h' is 1.15, not 1.12. In the frame where the clock is moving the clock ticks slowly by a factor of 1.15, so in 1.15 s moving at 0.5 c it travels a distance of 0.577 ls, not 0.5 ls. The Pythagorean theorem gives [itex]\sqrt{1^2+0.577^2}=1.15[/itex] ls, which contradicts your supposed "givens". The light travels 1.15 ls in 1.15 s which is c.
 
  • #130
DaleSpam said:
Your givens are wrong. h' is 1.15, not 1.12. In the frame where the clock is moving the clock ticks slowly by a factor of 1.15, so in 1.15 s moving at 0.5 c it travels a distance of 0.577 ls, not 0.5 ls. The Pythagorean theorem gives [itex]\sqrt{1^2+0.577^2}=1.15[/itex] ls, which contradicts your supposed "givens". The light travels 1.15 ls in 1.15 s which is c.
Oops I had overlooked that error in his "givens"! Thanks I'll also correct that for consistency.
 
  • #131
harrylin said:
BC=y=1 lightsecond = the distance between mirrors (measured with S) and this is a given.*

In addition, according to the second postulate (see above) the same must be true as measured with S'. There is no disagreement about the height of the train: y=y'= 1 lightsecond according to both S and S'. I hope that you see that. A light ray that is sent from C to B will take 1 s according to S'.*

Agreed, but noticed that this y is measured at the origin (it may not seem important, but it is, because this y will not be the y after the spatial rotation - where we must line up the x-axis of the observer to AB)



AC=x't'=0.5 lightseconds = distance traveled by the light clock in the x direction in 1 s.

*

TA=T'A= 0 = time of first recorded reflection, at both S and S' origins, which are both at A.

*

T'B≠1s. It is the time of the second reflection event as calculated in the rest frame S', based on the fact that CB= 1 light second. As a matter of fact, if according to S', two light flashes are sent simultaneously from A and C, the flash from C will arrive after 1s but the path length AB is longer, so it must take longer. Of course, it is assumed that the speed of light is the same in all directions; else no prediction is possible at all.*

The S' frame is the rest frame (not the moving frame), it is the measurements done from inside the light clock itself. So T'B = 1s



^RED mine.



The second postulate effectively says that you may set up an inertial reference system, pretend that it is at rest in the ether, and everything will work as if your assumption was correct...

No problem here, actually, if that isn't true my analysis fails. I don't know where is the misunderstanding coming from, though... I may need to clarify the problem further, since you have already shown that you misunderstood what I have been calling the primed frame (it is the rest frame).



Anyway, let's continue the -now trivial- analysis:

Yes!



T'B2=T'B + T'BA[/SUB = the moment a reflection signal (i.e.: scatter event) from B is seen at A in S, which is the time light takes to travel from the reflection event T'B to the observer at A. Of course, the trajectories AB and BA are the same, along h', and are assumed to take the same time.


Ah, the point of the setup in the first place - to check if this remains consistent.

If the observer at rest in A sent a flash of light from A to B, it would take the same time to reach B as the time the signal takes to reach A from B. BUT, we aren't dealing with that here. What we are seeking is not how fast light travels from A to B as seen from B or B to A from A. From A to B, it would seem to travel at c for an observer at B (and it would take 1.12 seconds to do so if h'=1.12 lightseconds). From B to A, it will seem to travel at c for the observer at A. But we are seeking how fast would the beam would seem to travel from A to B as seen from A. That's why we need the signal from B in the first place. As a matter of fact, our givens show that light takes 1s to go from A to B as seen from inside the lightclock (the primed frame), where the distance is 1 lightsecond.



This triangle represents the first two reflection events as seen in the moving frame S.

Pythagoras: Light path in S' AB = √(12 + 0.52)=1.12.

Thus one mirror is seen at A when T'=T=0 and the other at B when T=1, and T'=1.12[/QUOTE]

Actually, its the other way around: T' is the time in the rest frame - the time seen from the lightclock itself - and that is 1s.*

The light clock is going at 0.5c, as given by an onboard speedometer, useful in this setup to determine AC.*

Note that you did not draw the figure for S, which is simply vertical with y=y'.

*I didn't draw it because it is not necessary. Again, you may take that he only knows the primed times, sees the marked distance signs and receives the signal from B at a time TB.

AB = h' = 1.12

T'B2 = 2T'B = 2.24 s

VAB = 2h'/T'B2 = 1 lightsecond/s

You see right there! You just assumed the beam takes 1.12 seconds to go fram A to B as seen from A! You just assumed what had to be proven here. What you need to take into consideration is the primed time of event B and calculate the observed time for that event as seen from A, and then calculate the observed time from A to B.

In fact, it is quite useless to proceed if you don't master for example a classical analysis of the Michelson-Morley experiment.

But I do, and as I keep saying, this a different case. In MM we were always measuring the speed of received light, or light detected directly, which applies to the received signal from B in this setup. The postulate of the speed of light strictly states that c is absolute for the source and the observer regardlesd of motion, but that is clearly light that is in direct contact, in direct line, with either source or observer. This setup presents a novel configuration, which is light that doesn't reach the observer, and we have never performed any such experiment seeking the speed of distant light, as far as I know, so we have to be very careful with our assumptions and with our interpretation of the postulate of the speed of light. The goal of this setup is to calculate the speed of the beam, and clearly assumed this value so your analysis can't be proof that the speed of the beam as seen from A would actually be c.
 
  • #132
It seems that we were talking past each other, so I'll first only address the first two or three points:
altergnostic said:
^RED mine.
Once more: there is no spatial rotation here. Nothing rotates at all.
And once more, that was already explained in the link I gave, and of which you claimed to understand it (but evidently you don't):
https://www.physicsforums.com/showthread.php?t=574757

Please explain it in your own words before discussing further. :grumpy:

[..] I may need to clarify the problem further, since you have already shown that you misunderstood what I have been calling the primed frame (it is the rest frame).
Yes that is what I wrote: you clarified that S' is the rest frame, in which the light clock is moving; and S is the moving frame, in which the light clock is in rest. If you agree, then there is no misunderstanding about this.

Still it may be that the problem comes from confusion between frames, as you next write that "This setup presents a novel configuration, which is light that doesn't reach the observer" - however light always only reaches light detectors!

It may also come from a misunderstanding of the light postulate, or a combination of both, as you next write:

"The postulate of the speed of light strictly states that c is absolute for the source and the observer regardlesd of motion".

That is certainly not the second postulate. Did you study the Michelson-Morley calculation? And if so, do you understand it? Then please explain it.

PS. This forum is meant to explain how SR works. It is not meant to "prove" a theory. As a matter of fact, such a thing is impossible!
 
Last edited:
  • #133
altergnostic said:
The goal of this setup is to calculate the speed of the beam
This stated goal is mathematically impossible. By the Pythagorean theorem for any signal speed C we have the geometrical relationship [itex]C^2 t^2 = y^2 + v^2 t^2[/itex]. If you fix y and v then assuming a value for t is the same as assuming a value for C. Here you are incorrectly assuming that t=0.5, which is equivalent to assuming that C≠c.

altergnostic said:
and clearly assumed this value so your analysis can't be proof that the speed of the beam as seen from A would actually be c.
Yes. We ASSUME that the speed of the beam is c. That is what a postulate is, a fundamental assumption of the theory. If you are assuming anything contrary (which you are) then you are not doing SR. From that assumption we can then calculate the value of t using the relationship above.

There is no mathematical proof of the validity of the assumption (you can't mathematically prove postulates or axioms, by definition), however an enormous body of experimental evidence supports the assumption and contradicts your alternative assumption.
 
  • #134
altergnostic said:
The goal of this setup is to calculate the speed of the beam

In my analysis, I *calculated* (*not* assumed) the speed of the light pulse traveling between events B1a and D2, neither of which are co-located with the observer, so this is a "distant light" beam by your definition. Its speed is *calculated* to be c. So my analysis meets your goal. What's the problem?
 
  • #135
PeterDonis said:
No, he can't. He has to get light signals from them, just like from everything else that's not at his spatial location. Putting km markers everywhere doesn't magically transmit information instantaneously.

I don't mean that he sees anything instantaneously, this is just how he will measure anything. Length contraction applies to vt, but the markers are just distances and they are at rest relative to the observer. These markers belong to both frames, and they are not being measured by any of them. You could call them x'' or something. They are given and are not being measured in the process. Usually, we are given V and find the distance from that, but since we are seeking V, I have to give L somehow. A given V contains a given L which is the same for both frames, you see. This is the given distance. The time it would take for light to go from A to B, as seen from B, is the same as from B to A seen from A, so this distance must be the same and it is not being measured by any of the frames. It is a given just like the distance implied in the given V from most SR thought problems. If you can have the same velocity between two frames, you already have an equivalent distance between two frames, over the standard unit of time.

In any case, no matter how he gets the numbers, you can't mix numbers from different frames and expect to get meaningful answers. Any valid calculation of the relative velocity has to use either an unprimed distance and corresponding unprimed time, or a primed distance and a corresponding primed time. Using a primed distance and unprimed time is just as meaningless as using an unprimed distance and a primed time.

See above.

For an object that moves at less than the speed of light, the velocity of that same object is different in different frames, yes. But that doesn't apply to light; the velocity of a light beam is c in every frame (assuming inertial frames and flat spacetime). I've explicitly proven that to be the case in your scenarios.

You have not proven it, you have assumed it, since the operation you developed is not the same as you would if the beam was a projectile. And you simply stated that it must be different because we are supposed to know that the beam is moving at c from A to B as seen from A, and that's not a proof, that is an assumption. You claim it is an assumption backed up by numerous experiments, but not a single experiment in history has measured the speed of light that does not reach a detector directly, you have to aknowledge this. I state again that c applies to light directly detected, light that reaches and "touches" the observer.

Also, you have claimed that the *relative* velocity of two frames is different in one frame than in the other; this is the basis of your claim that you don't know which gamma factor to use. The fact that the velocity of the same object (if it moves slower than light) is different in different frames is irrelevant to that claim, because here you are not comparing the velocity of the same object in different frames; you are comparing the velocities of *different* objects in different frames. You are comparing the velocity of the train in the observer frame, with the velocity of the observer in the train frame. Those two velocities have the same magnitude, but opposite directions; I've proven that.

That is correct, and it is obvious anyway. But in this case we ARE measuring the speed of the same "object" in different frames: the setup is meant to give us the oportunity to measure the speed of the beam from both frames.

And I repeat, this is *not* an assumption; it is a proven fact. I've proven it in my previous posts. You can prove that all light beams travel at c using only facts about the relative motion of the light clock and the observer (or the train and the ground), without making any assumptions about the speed of the light beam.

You haven't shown this at all, Peter. If you carefully check your analysis you will see that you assumed the observed speed of the beam seen from A is c, not shown it.

[/QUOTE]Because the beam is moving at c, and the projectile is moving slower than c. The two cases are different. That's just a fact about how relativistic spacetime works. The only assumption I can see that is worth mentioning here is the isotropy of space (since that's really what underlies the claim that the velocity of the observer relative to the clock is equal in magnitude and opposite in direction to the velocity of the clock relative to the observer). That assumption is supported by a massive amount of evidence, so I don't see the point of questioning it here.[/QUOTE]

Now you do assume it is an assumption. The evidence shows the speed of light is c for directly detected light, as I said above. I am not aware of any experiments that allow us to measure the speed of non-observed light, in the sense that this is light that doesn't reach a detector directly.

By using known facts about the speed of the light clock relative to him to establish the coordinates of the relevant events in the unprimed frame, independently of the speed of the light signals that travel between those events.

Again, assumptions. And not backed up by evidence at all - evidence shows that detected light moves at c. No one can propose that the signal from B to A moves at any other speed than c, since that would contradict every single experiment ever done to measure the speed of light. Since no experiment has ever been performed with distant and indirectly detected light, the assumption that the beam moves at c is not supported by any evidence whatsoever. Not for, nor against.



Well, of course. Why not?

Because you must calculate the speed of the beam before assuming it is c, since that is the point of the setup in the first place: the variables given allows us to measure the speed of the beam.



I don't know what "calculating the speed of the beam from the signals" means. I'm calculating the speed of the beam from knowledge of the distance and time intervals between two known events on the beam's worldline.

It means giving c to the signal and plotting the observed times if events A and B against the given L.

Then I can re-write all the formulas in terms of the unknown gamma factor for the relative velocity, and I will *still* get the same answer. All the factors of gamma end up canceling out. You should be able to see that from my analysis, but if not, I can re-write it this way (although you really should be able to do that yourself). In fact, I actually do re-write the formula for the velocity of the light beam in the light clock this way (for comparison with the velocity of a projectile in a "projectile clock") below.

But you didn't do it relative to A, or "as the observer at A would see it", as I said above.

Yes. But you are aware, aren't you, that this implies tB = gamma, right?

You can't assume that since you aren't given the speed of the beam. What V do you insert on gamma?


Is this supposed to be a distance in the unprimed frame? If so, how do you know it is consistent with the values for tA, t'A, and t'B that you just gave? If you start out with an inconsistent set of premises, of course you're going to get a meaningless answer.

It is a distance measured locally prior to the experiment. None of the frames are measuring it, they are given it. Just like the given distance implied if you were given V.

I don't see how this follows at all. AC is supposed to be the distance, in the unprimed frame, that the light clock source/detector travels during the time of flight of the beam, correct? In that case, it should be gamma, not gamma primed, because the time of flight of the beam, in the unprimed frame, is tB = gamma (see above).

This is a typo, it was meant to be BC, sorry for this. But I guess you did notice this honest mistake, didn't you? The diagram shows clearly where the y' distance stands for.

So already I have spotted two wrong premises in your argument; no wonder you're getting erroneous answers.

You spotted a typo and a given that is also given if you were given V, so no wrong premises here.


Yes. But again, you are aware that this implies tB = 2 gamma, right?

Not really. First you have to find the velocity, otherwise what V do you plug into gamma? It may be, it may not be, you have to show that it is, the setup is supposed to allow us to calculate the speed. Than you can show what is the value for gamma.

Same comment as above. This value of AB is not consistent with the above, and AC is 2 gamma, not gamma primed, or even 2 gamma primed, which is what you should have written by the same reasoning as the above--t'B is twice as large, so AB should be twice as large as well since the light clock spends twice as much time traveling as before. You are aware, aren't you, that using a projectile traveling at 0.5c, instead of a light beam traveling at c, changes the geometry of your triangle diagram, right? It's *not* the same triangle.
The same typo. Also, although you are right and the triangle does change, the hypotenuse is still AB, the distances are still given and the setup is the same. I haven't done the calculations to find the values, though, but it should be clear that the setup is equivalent, and so the operation should be, else you lose consistency and have to prove why this should be so. Since your reason is experimental evidence, I remind you again that no experiment has measured the speed of undetected or indirectly observed light or distant light or light that does not reach the observer/detector.


I've already done the light beam case in my previous analysis, but perhaps it would help to do a similar analysis on the "projectile clock" case; we'll assume a projectile traveling at 0.5c in the primed frame, in the direction perpendicular to the relative motion of the light clock and the observer in that frame. I'll orient the x-axis in the direction the projectile travels, and the y-axis in the direction of the relative motion of the "projectile clock" and the observer. (Yes, I know you think that the x-axis has to point directly along the path to where the mirror will be in the unprimed frame when the projectile hits it. I've already shown that that doesn't matter; I don't think it needs to be shown again. But rotating the spatial axes will be easy enough after the analysis I'm about to show, if you really insist on seeing it done that way.)

We are given the following event coordinates:

[tex]A0 = D0 = A0' = D0' = (0, 0, 0)[/tex]

I assume that D0 here stands for A0 right? You are specifying that they are the same point in the projectile-clock, right? I'm asking this because D is really unecessary after all, we can focus on A, B and C and find what we seek from that.

[tex]B1a' = B1b' = (2, 1, 0)[/tex]
[tex]D2' = (4, 0, 0)[/tex]

This assumes a "projectile" traveling at 0.5c in the primed frame, in the x-direction.

The transformation equations are (I'm writing them now in terms of an unknown gamma factor, since you are now saying we don't know the relative velocity of the light clock and the observer):

I'm saying we don't know the PROJECTILE velocity. We don't need the clock velocity at all, since we are given the basic coordinates that must compose the relative velocity - which is the unknown we seek.

Unprimed to Primed:

[tex]t' = \gamma ( t - v y )[/tex]
[tex]x' = x[/tex]

The vy term should be vAB, since the speed from A to B is not the same from A to C - that would be the speed of the clock, but we are transforming the projectile, not the clock.

[tex]y' = \gamma ( y - v t )[/tex]

Primed to Unprimed:

[tex]t = \gamma ( t' + v y' )[/tex]
[tex]x = x'[/tex]
See above

[tex]y = \gamma ( y' + v t' )[/tex]

This yields the following coordinates for events in the unprimed frame:

[tex]B1a = B1b = ( 2 \gamma, 1, 2 \gamma v )[/tex]
[tex]D2 = ( 4 \gamma, 0, 4 \gamma v )[/tex]

The velocity of the projectile in the unprimed frame on each leg can then be computed like this:

[tex]v_{projectile} = \frac{\sqrt{\Delta x^2 + \Delta y^2}}{\Delta t} = \frac{\sqrt{1 + 4 \gamma^2 v^2}}{2 \gamma} = \frac{1}{2} \sqrt{(1 - v^2) \left( 1 + \frac{4 v^2}{1 - v^2} \right)} = \frac{1}{2} \sqrt{\frac{(1 - v^2)(1 - v^2 + 4 v^2)}{1 - v^2}} = \frac{1}{2} \sqrt{1 + 3 v^2}[/tex]

This will be somewhere between 1/2, which is the velocity of the projectile in the primed frame, and 1 (but always less than 1 for v < 1); but it is only *equal* to 1/2 for v = 0, so you are correct that the velocity of the projectile in a "projectile clock" *does* change if you change frames.

Yes, but that is the velocity you should plug into gamma (except in the case where the projectile and the clock motion were in the same line). You must do a vector addition and find the speed of the projectile from the unprimed frame at some point, if you want to use a velocity value, but you don't a variable v at all to solve. So gamma is compromised.

I could leave it to you to spot the difference between this and the case of the light clock, but I suppose I'll go ahead and give it; the corresponding formula from my previous analysis would be (written with an unknown gamma factor to make it clear how it drops out of the analysis):

[tex]v_{beam} = \frac{\sqrt{\Delta x^2 + \Delta y^2}}{\Delta t} = \frac{\sqrt{1 + \gamma^2 v^2}}{\gamma} = \sqrt{(1 - v^2) \left( 1 + \frac{v^2}{1 - v^2} \right)} = \sqrt{\frac{(1 - v^2)(1 - v^2 + v^2)}{1 - v^2}} = 1[/tex]

Again, how do you know what V to use, since you aren't given any? You must find a way to solve without it, since I have only given the L component of the relative velocity. You have to find the observed velocity, since that's what we are seeking. As it is, you have two different velicities in your equations, and you are given none, so the only thing I can conclude from your analysis (if it is correct, and I don't think it is for the reasons above) is that the setup doesn't give enough information to solve. But I know this isn't your conclusion.

As you can see, this formula always gives 1, regardless of v (as long as v < 1). So unlike the projectile case, the speed of a light beam in a light clock does *not* change when you change frames.

This is only so because you assume Vbeam = c in both frames. You assume what has to be proven.

PeterDonis said:
Just to put some icing on the cake, here's an even more general formula which works for arbitrary "projectile" velocities (thus including the case of the light beam as the "projectile" as well).

We have a clock using a "projectile" that travels between a source/detector and a reflector. The projectile's velocity is [itex]v_p'[/itex] in the rest frame of the clock. We are given the following event coordinates (the projectile travels in the x direction):

[tex]A0 = D0 = A0' = D0' = (0, 0, 0)[/tex]
[tex]B1a' = B1b' = (1 / v_p', 1, 0)[/tex]
[tex]D2' = (2 / v_p', 0, 0)[/tex]

The transformation equations are as before, and they yield the following coordinates for events in the unprimed frame:

[tex]B1a = B1b = ( \gamma / v_p', 1, \gamma v / v_p')[/tex]
[tex]D2 = ( 2 \gamma / v_p', 0, 2 \gamma v / v_p')[/tex]

The velocity of the projectile in the unprimed frame on each leg is then:

[tex]v_p = \frac{\sqrt{\Delta x^2 + \Delta y^2}}{\Delta t} = \frac{\sqrt{1 + \gamma^2 v^2 / v_p'^2}}{\gamma / v_p' } = v_p' \sqrt{(1 - v^2) \left( 1 + \frac{v^2}{v_p'^2 (1 - v^2 )} \right)} = v_p' \sqrt{\frac{(1 - v^2)(1 - v^2 + v^2 / v_p'^2)}{1 - v^2}} = v_p' \sqrt{1 + v^2 \left( \frac{1}{v_p'^2} - 1 \right)}[/tex]

For [itex]0 < v_p' < 1[/itex], this gives [itex]v_p' < v_p < 1[/itex] for any [itex]0 < v < 1[/itex]. For [itex]v_p' = 1[/itex], this gives [itex]v_p = 1[/itex] for any [itex]0 < v < 1[/itex]. QED.

I prefer this analysis over the previous one, but the problem remains: you are not given any v, you must find another way to solve (which I believe is only possible if you can find vp - the velocity observed from A, in my diagram), and that was the purpose of my setup, and the reason I gave the L component for the relative velocity(s).

If you still have trouble with my given x' and y' - maybe a better form would be X and Y since they are given for both frames - ask yourself what L is implied in the relative V.
 
  • #136
DaleSpam said:
Your givens are wrong. h' is 1.15, not 1.12. In the frame where the clock is moving the clock ticks slowly by a factor of 1.15, so in 1.15 s moving at 0.5 c it travels a distance of 0.577 ls, not 0.5 ls. The Pythagorean theorem gives [itex]\sqrt{1^2+0.577^2}=1.15[/itex] ls, which contradicts your supposed "givens". The light travels 1.15 ls in 1.15 s which is c.

h' is not being measured, it is given, it is supposed to be the constituent L of the relative V that is usually given in SR problems, which we are not given in this setup and is the unknown we seek.
 
  • #137
harrylin said:
Oops I had overlooked that error in his "givens"! Thanks I'll also correct that for consistency.

Please see my answer above and also my reply to Peter.
 
  • #138
altergnostic said:
Please see my answer above and also my reply to Peter.
Your "givens" are according to SR selfcontradictory; when I corrected it, I showed this by two methods of calculation and I explained it with words. Note that both my approaches differed a little from that of Dalespam and PeterDonis; I only used your input.
 
Last edited:
  • #139
harrylin said:
It seems that we were talking past each other, so I'll first only address the first two or three points:

Yes, it is getting harder by the relative minute to keep up with so many posts.

Once more: there is no spatial rotation here. Nothing rotates at all.
And once more, that was already explained in the link I gave, and of which you claimed to understand it (but evidently you don't):
https://www.physicsforums.com/showthread.php?t=574757

Please explain it in your own words before discussing further. :grumpy:

The spatial rotation refers to the observer line of sight. I propose that he places his x-axis in line with AB so he can give all the motion to beam and solve without knowledge of the clock's velocity.

From the link, I will comment only upon the same issue I have been discussing here: the beam leaves A and reaches B in the rest frame (the frame that is at rest relative to the clock) and moves at c as seen from a detector at B. From the perspective of an observer that sees the clock in relative motion, the brings no data and he can't say anything about its observed velocity, since it is not observed. By including a signal sent from B to A at the moment the beam reaches B, the observer will have the observed time for that event and may determine time of the event as recorded on the detector by taking into consideration the observed time and the predetermined distance between him and the detector at B, just like the standard relative V implies a given distance/unit of time.

Yes that is what I wrote: you clarified that S' is the rest frame, in which the light clock is moving; and S is the moving frame, in which the light clock is in rest. If you agree, then there is no misunderstanding about this.*

Maybe my terminology is incorrect? The rest frame is meant to be the frame at rest relative to the clock. The moving frame is supposed to be the observer who sees the moving clock.

Still it may be that the problem comes from confusion between frames, as you next write that "This setup presents a novel configuration, which is light that doesn't reach the observer" - however light always only reaches light detectors!*

The beam never reaches the observer at A. Only the signal from B does. He then marks the observed time of event B and plots it against the given L. He can then find the time of event B as marked on the detector's clock by subtracting the time it takes light to cross the distance between the coordinates of the event at B from the observed time. From that, he can calculate the distance from source to receiver as seen from the clock itself by transforming the calculated speed of the beam into c (the speed of light as seen locally, which is the speed supported by a huge amount of evidence). So you see, we only need to know the predetermined distance from A to B, the given time of detection as seen from the clock and the observed time of event B, to figure everything else

It may also come from a misunderstanding of the light postulate, or a combination of both, as you next write:

"The postulate of the speed of light strictly states that c is absolute for the source and the observer regardlesd of motion".*

That is certainly not the second postulate. Did you study the Michelson-Morley calculation? And if so, do you understand it? Then please explain it.

The interferometer was supposed to test the existence of the aether and find a variance of the speed of light relative to the ether. The setup was built so we had perdicular light paths. As the apparatus (and the Earth) revolved, no fringe displacement was detected (actally, no significant displacement). So it was concluded that the speed of light was constant regardless of the orientation of the beams or the detectors relative to an absolute frame. Notice that in the equations applied to this experiment, V was the relative velocity wrt the aether, so there was only one possible V, as the aether was absolute. Since then, the speed if light was measured with ever increasing accuracy, always in the same manner: by noting the return time if light as it went forth and back a specified distance. The time of detection after emission is always proportional to c. This is exactly the premise of my setup: light can't be detected to move at any V other than c. Just notice that the beam is not detected by A, only the signal from B is. There's no return, no detection, nothing that relates the path of the beam with the experiments that tested the speed of light.

PS. This forum is meant to explain how SR works. It is not meant to "prove" a theory. As a matter of fact, such a thing is impossible!

This is no theory other than SR itself, since the same postulates apply to detected light, consistent with every experiment ever done. A correction to an incorrect or incomplete diagram or a new thought problem that does not contradict but expands theory is not a new theory, since it is built upon the same postulates. This is merely the outcome of a realisation that this thought problem has never been done before and that the postulate of the speed of light applies to source and receiver, but not necessarily to a non-receiver. This setup is meant to analyse this realisation, keeping all aspects of Einstein's original theory intact and remaining consistent with all available empirical data.

It is the preconception that the postulate of the speed of light also applies to undetected light that keeps you from aknowledging the possibility I intend to discuss here.
Under close inspection, you may realize that neither the postulate nor the evidence are in contradiction with my conclusions, but that it is a necessary outcome of relativity. It is only ligical, if light reaches us at a constant speed, distance events will be seen at a later time than the time of the event determined locally (for an observer in close proximity and at rest relative to the event). Hence, if the observed time is delayed and distances are given, the observed speed must be smaller.

To realize this is consistent with SR may be difficult, but it must be, since the conclusion is strictly dependant upon the acceptance of the constancy of the speed of any received or detected light.
 
  • #140
harrylin said:
Your "givens" are according to SR selfcontradictory; when I corrected it, I showed this by two methods of calculation and I explained it with words. Note that both my approaches differed a little from that of Dalespam and PeterDonis; I only used your input.

Why are they self contradictory? If I walked from A to B and measured the distance, wouldn't this distance be the same you would measure by walking from B to A? This is as symmetrical as any relative velocity, as it must be. If there is no symmetric distance between frames, there can be no symmetric speed. Do you disagree? If so, please explain why and show me how to determine relative velocity without a equally symmetric relative distance.

Are there other givens you still disagree with?
 
<h2>1. What is the paradoxical light clock problem in special relativity analysis?</h2><p>The paradoxical light clock problem is a thought experiment used to illustrate the effects of time dilation in special relativity. It involves two identical clocks, one stationary and one moving at a high speed, with a beam of light bouncing between them. According to the theory of special relativity, the moving clock will appear to tick slower than the stationary clock, leading to a paradox where the moving clock appears to be both ticking slower and faster at the same time.</p><h2>2. How does special relativity explain the paradoxical light clock problem?</h2><p>Special relativity explains the paradoxical light clock problem by taking into account the concept of time dilation, which states that time moves slower for objects that are moving at high speeds. This means that the moving clock in the thought experiment will actually tick slower than the stationary clock, and the apparent paradox is resolved.</p><h2>3. What are some real-world examples of time dilation in special relativity?</h2><p>Some real-world examples of time dilation in special relativity include the Global Positioning System (GPS) and particle accelerators. GPS satellites orbit the Earth at high speeds, causing their onboard clocks to experience time dilation. This must be taken into account for accurate GPS calculations. In particle accelerators, particles are accelerated to near the speed of light, causing them to experience time dilation.</p><h2>4. How does the paradoxical light clock problem impact our understanding of time and space?</h2><p>The paradoxical light clock problem, and the theory of special relativity in general, has greatly impacted our understanding of time and space. It has shown that time is not absolute, but rather is relative to the observer's frame of reference. It has also led to the concept of space-time, where time and space are interconnected and can be affected by factors such as gravity and velocity.</p><h2>5. Are there any unresolved questions or criticisms of the paradoxical light clock problem in special relativity analysis?</h2><p>While the paradoxical light clock problem has been widely accepted and used to support the theory of special relativity, there are still some unresolved questions and criticisms. Some argue that the thought experiment oversimplifies the concept of time dilation and does not take into account other factors such as length contraction. Others question the validity of the assumptions made in the thought experiment. However, the paradoxical light clock problem remains a valuable tool in understanding the effects of time dilation in special relativity.</p>

1. What is the paradoxical light clock problem in special relativity analysis?

The paradoxical light clock problem is a thought experiment used to illustrate the effects of time dilation in special relativity. It involves two identical clocks, one stationary and one moving at a high speed, with a beam of light bouncing between them. According to the theory of special relativity, the moving clock will appear to tick slower than the stationary clock, leading to a paradox where the moving clock appears to be both ticking slower and faster at the same time.

2. How does special relativity explain the paradoxical light clock problem?

Special relativity explains the paradoxical light clock problem by taking into account the concept of time dilation, which states that time moves slower for objects that are moving at high speeds. This means that the moving clock in the thought experiment will actually tick slower than the stationary clock, and the apparent paradox is resolved.

3. What are some real-world examples of time dilation in special relativity?

Some real-world examples of time dilation in special relativity include the Global Positioning System (GPS) and particle accelerators. GPS satellites orbit the Earth at high speeds, causing their onboard clocks to experience time dilation. This must be taken into account for accurate GPS calculations. In particle accelerators, particles are accelerated to near the speed of light, causing them to experience time dilation.

4. How does the paradoxical light clock problem impact our understanding of time and space?

The paradoxical light clock problem, and the theory of special relativity in general, has greatly impacted our understanding of time and space. It has shown that time is not absolute, but rather is relative to the observer's frame of reference. It has also led to the concept of space-time, where time and space are interconnected and can be affected by factors such as gravity and velocity.

5. Are there any unresolved questions or criticisms of the paradoxical light clock problem in special relativity analysis?

While the paradoxical light clock problem has been widely accepted and used to support the theory of special relativity, there are still some unresolved questions and criticisms. Some argue that the thought experiment oversimplifies the concept of time dilation and does not take into account other factors such as length contraction. Others question the validity of the assumptions made in the thought experiment. However, the paradoxical light clock problem remains a valuable tool in understanding the effects of time dilation in special relativity.

Similar threads

  • Special and General Relativity
Replies
7
Views
951
  • Special and General Relativity
Replies
14
Views
583
  • Special and General Relativity
Replies
12
Views
1K
  • Special and General Relativity
Replies
15
Views
317
  • Special and General Relativity
Replies
16
Views
597
  • Special and General Relativity
Replies
20
Views
1K
  • Special and General Relativity
Replies
6
Views
2K
  • Special and General Relativity
2
Replies
65
Views
3K
  • Special and General Relativity
Replies
14
Views
1K
  • Special and General Relativity
Replies
21
Views
1K
Back
Top